2011 AMC 8 Problems/Problem 4

Revision as of 18:03, 25 November 2011 by Yrushi (talk | contribs) (Problem 4)
(diff) ← Older revision | Latest revision (diff) | Newer revision → (diff)

Here is a list of the numbers of fish that Tyler caught in nine outings last summer: \[2,0,1,3,0,3,3,1,2.\] Which statement about the mean, median, and mode is true?

$\text{(A)} \text{median} < \text{mean} < \text{mode} \qquad \text{(B)} \text{mean} < \text{mode} < \text{median} \\ \\ \text{(C)} \text{mean} < \text{median} < \text{mode} \qquad \text{(D)} \text{median} < \text{mode} < \text{mean} \\ \\ \text{(E)} \text{mode} < \text{median} < \text{mean}$

Solution

See Also

2011 AMC 8 (ProblemsAnswer KeyResources)
Preceded by
Problem 3
Followed by
Problem 5
1 2 3 4 5 6 7 8 9 10 11 12 13 14 15 16 17 18 19 20 21 22 23 24 25
All AJHSME/AMC 8 Problems and Solutions